Last visit was: 27 Apr 2024, 10:00 It is currently 27 Apr 2024, 10:00

Close
GMAT Club Daily Prep
Thank you for using the timer - this advanced tool can estimate your performance and suggest more practice questions. We have subscribed you to Daily Prep Questions via email.

Customized
for You

we will pick new questions that match your level based on your Timer History

Track
Your Progress

every week, we’ll send you an estimated GMAT score based on your performance

Practice
Pays

we will pick new questions that match your level based on your Timer History
Not interested in getting valuable practice questions and articles delivered to your email? No problem, unsubscribe here.
Close
Request Expert Reply
Confirm Cancel
SORT BY:
Kudos
Tags:
Show Tags
Hide Tags
User avatar
Director
Director
Joined: 26 Apr 2004
Posts: 548
Own Kudos [?]: 4449 [11]
Given Kudos: 0
Location: Taiwan
Send PM
User avatar
Director
Director
Joined: 30 Sep 2004
Posts: 687
Own Kudos [?]: 1448 [1]
Given Kudos: 0
Location: Germany
Send PM
Intern
Intern
Joined: 24 Jun 2018
Posts: 29
Own Kudos [?]: 66 [1]
Given Kudos: 31
Send PM
User avatar
Director
Director
Joined: 18 Nov 2004
Posts: 679
Own Kudos [?]: 198 [0]
Given Kudos: 0
Send PM
Re: A museum directors, in order to finance expensive new acquisitions, di [#permalink]
"E" is best. Just becose something turned out to be as predicted doesn't tell us that the original argument was valid. There may have been other causes for the director to sell those arts.
User avatar
Intern
Intern
Joined: 05 Jun 2004
Posts: 5
Own Kudos [?]: 1 [0]
Given Kudos: 0
Location: India
Send PM
Re: A museum directors, in order to finance expensive new acquisitions, di [#permalink]
E it is..

Conclusion - The museum pieces sold were first-rate
Facts - It sold at thrice the prices sold by the museum director.

But then the museum pieces could have sold at a higher price bcoz of
other reasons...(speculation is a good one!) and not bcoz the pieces were
of the best quality.

A is wrong coz the argument doesn't mention anything about the buyer's opinion about the museum pieces. It just says that the museum pieces
sold for 3 times the previous price.
avatar
Intern
Intern
Joined: 28 Sep 2014
Posts: 32
Own Kudos [?]: 10 [0]
Given Kudos: 2
Location: United States
GPA: 3.7
Send PM
Re: A museum directors, in order to finance expensive new acquisitions, di [#permalink]
I understand that (E) is correct, but I don't understand why (D) is incorrect. The argument does appear to rely on "somewhat speculative claims about a future state of affairs" since the high prices were settled at a time later than when the "experts" remark that the museum had lost first-rate pieces.
Intern
Intern
Joined: 24 Jul 2018
Posts: 41
Own Kudos [?]: 37 [0]
Given Kudos: 21
Location: India
Schools: IMD '21
GPA: 4
Send PM
Re: A museum directors, in order to finance expensive new acquisitions, di [#permalink]
chunjuwu wrote:
A museum directors, in order to finance expensive new acquisitions, discreetly sold some paintings by major artists. All of them were paintings that the director privately considered inferior.
Critics roundly condemned the sale, charging that the museum had lost first-rate pieces, thereby violating its duty as a trustee of art for future generations. A few months after being sold by the museum, those paintings were resold, in an otherwise stagnant art market, at two to three times the prices paid to the museum. Clearly, these prices settle the issue, since they demonstrate the correctness of the critics’ evaluation.

The reasoning in the argument is vulnerable to the criticism that the argument does which one of the following?

(A) It concludes that a certain opinion is correct on the grounds that it is held by more people than hold the opposing views.

(B) It rejects the judgment of the experts in an area in which there is no better guide to the truth than expert judgment.

(C) It rejects a proven means of accomplishing an objective without offering any alternative means of accomplishing that objective.

(D) It bases a firm conclusion about a state of affairs in the present on somewhat speculative claims about a future state of affairs.

(E) It bases its conclusion on facts that could, in the given situation, have resulted from causes other than those presupposed by the argument.


Can anybody please explain the QUESTION in details? I am unable to understand the question itself. :roll:
Manager
Manager
Joined: 21 Jul 2018
Posts: 153
Own Kudos [?]: 434 [0]
Given Kudos: 80
Location: United States
Concentration: General Management, Social Entrepreneurship
Send PM
Re: A museum directors, in order to finance expensive new acquisitions, di [#permalink]
Ritwick91 wrote:

Can anybody please explain the QUESTION in details? I am unable to understand the question itself. :roll:


Ritwick91 Happy to help.

Quote:
A museum directors, in order to finance expensive new acquisitions, discreetly sold some paintings by major artists. All of them were paintings that the director privately considered inferior. Critics roundly condemned the sale, charging that the museum had lost first-rate pieces, thereby violating its duty as a trustee of art for future generations. A few months after being sold by the museum, those paintings were resold, in an otherwise stagnant art market, at two to three times the prices paid to the museum. Clearly, these prices settle the issue, since they demonstrate the correctness of the critics’s evaluation.


The reasoning in the argument is vulnerable to the criticism that the argument does which one of the following?

The background context of this argument is:

[*] Museum directors need money so they sold the "not so great" paintings.
[*] Critics were mad about this move, saying they lost "first-rate pieces".
[*] Painting got resold "at two to three times the price paid" (Oh, sounds the critics are going to equate price to quality!)
[*] CLEARLY, "prices settle the issue".

The question stem asks us to ATTACK the "reasoning in the argument." What does the argument say? The critic’s argue that because the resale price of this art was super higher, therefore, their evaluation of that art as “first-rate” must be true. Their faulty belief/conclusion is price is indicative of quality. A flaw in this reasoning is that the critics assume is the cause and effect relationship. If this was a weaken question, a possible answer choice would likely be an “incomplete information answer” or one that sheds evidence that price doesn’t indicate quality. Or that shares the possibility that art prices are heavily influenced by the economy.

Quote:
(A) It concludes that a certain opinion is correct on the grounds that it is held by more people than hold the opposing views.

This answer choice tells us that popular opinions are correct. We don’t know if the critics outnumber of the directors. Also, we are certain this is a causality error!

Quote:
(B) It rejects the judgment of the experts in an area in which there is no better guide to the truth than expert judgment.

Is museum director or the critic the expert judgement? Hm, are we sure there is “no better guide”?

Quote:
(C) It rejects a proven means of accomplishing an objective without offering any alternative means of accomplishing that objective.

There was not objective to be accomplished.

Quote:
(D) It bases a firm conclusion about a state of affairs in the present on somewhat speculative claims about a future state of affairs.

We never discussed a present v. future state of affairs.

Quote:
(E) It bases its conclusion on facts that could, in the given situation, have resulted from causes other than those presupposed by the argument.

Ah, this sounds like our pre-think. Yes, what if the market influences prices?
Intern
Intern
Joined: 01 Oct 2022
Posts: 2
Own Kudos [?]: 0 [0]
Given Kudos: 11
Send PM
Re: A museum directors, in order to finance expensive new acquisitions, di [#permalink]
kelloggcookiemonster wrote:
I understand that (E) is correct, but I don't understand why (D) is incorrect. The argument does appear to rely on "somewhat speculative claims about a future state of affairs" since the high prices were settled at a time later than when the "experts" remark that the museum had lost first-rate pieces.



so even i was a bit stuck between D AND E but the word speculation is bothering me as in the pssg. it states that it was sold at a higher rate and it i could be wrong here but while putting D there will be many variables also then i marked D.
hope this hepls
CEO
CEO
Joined: 07 Mar 2019
Posts: 2555
Own Kudos [?]: 1813 [0]
Given Kudos: 763
Location: India
WE:Sales (Energy and Utilities)
Send PM
Re: A museum directors, in order to finance expensive new acquisitions, di [#permalink]
A museum directors, in order to finance expensive new acquisitions, discreetly sold some paintings by major artists. All of them were paintings that the director privately considered inferior. Critics roundly condemned the sale, charging that the museum had lost first-rate pieces, thereby violating its duty as a trustee of art for future generations. A few months after being sold by the museum, those paintings were resold, in an otherwise stagnant art market, at two to three times the prices paid to the museum. Clearly, these prices settle the issue, since they demonstrate the correctness of the critics’ evaluation.

The reasoning in the argument is vulnerable to the criticism that the argument does which one of the following?

(A) It concludes that a certain opinion is correct on the grounds that it is held by more people than hold the opposing views. - WRONG. Not held by more people neither it is said to hold opposing views.

(B) It rejects the judgment of the experts in an area in which there is no better guide to the truth than expert judgment. - WRONG. Opposite to what it did and also the reasoning offered here is wrong.

(C) It rejects a proven means of accomplishing an objective without offering any alternative means of accomplishing that objective. - WRONG. Like B only.

(D) It bases a firm conclusion about a state of affairs in the present on somewhat speculative claims about a future state of affairs. - WRONG. Both speculation and future state of affairs is wrong.

(E) It bases its conclusion on facts that could, in the given situation, have resulted from causes other than those presupposed by the argument. - CORRECT. The claim's reasoning is not necessarily true. Being sold at 2-3 times does not makes critics' argument correct. Though even POE helps.

The passage is easy to understand but the options make it difficult.

Answer E.
GMAT Club Bot
Re: A museum directors, in order to finance expensive new acquisitions, di [#permalink]
Moderators:
GMAT Club Verbal Expert
6923 posts
GMAT Club Verbal Expert
238 posts
CR Forum Moderator
832 posts

Powered by phpBB © phpBB Group | Emoji artwork provided by EmojiOne